without negating
i did not read this question stem as a sufficient with a necessary premise so i missed eliminatin...
Elizabeth25 on May 7 at 08:01PM
  • June 1993 LSAT
  • SEC4
  • Q10
1
Reply
Explanation
Can someone explain why B is the answer and not A?
YAS1 on February 28, 2023
  • June 1993 LSAT
  • SEC4
  • Q9
1
Reply
Answer Choice and explanation
The explanation is treating this as a Strengthen with Necessary question but its a Strengthen wit...
AndrewArabie on December 8, 2022
  • June 1993 LSAT
  • SEC4
  • Q21
1
Reply
Explanation
Can someone explain why the other answers were wrong I got it correct but didn't confidently elim...
Jasmin1 on October 19, 2022
  • June 1993 LSAT
  • SEC4
  • Q25
2
Replies
b v E
I got the answer correct mainly through process of elimination, and I can understand why E is rig...
AllisonJ on October 19, 2022
  • June 1993 LSAT
  • SEC4
  • Q23
1
Reply
Wrong category
This question is a Strengthen with Necessary Premise, but it is included in the Strengthen with S...
kgbnd on October 19, 2022
  • June 1993 LSAT
  • SEC4
  • Q21
1
Reply
Answer Choice C
Would C just be wrong because "reputation" is not strong enough?
Sallyanne-Tejan on July 16, 2022
  • June 1993 LSAT
  • SEC4
  • Q12
1
Reply
How does D tear the argument apart?
This has nothing to do with the argument.
Tyler808 on July 9, 2022
  • June 1993 LSAT
  • SEC4
  • Q18
2
Replies
Why is E incorrect?
Why is E incorrect? Thanks
jingjingxiao11111@gmail.com on February 11, 2022
  • June 1993 LSAT
  • SEC4
  • Q24
6
Replies
why b over d?
please explain why b over d
michellesheinker@gmail.com on October 28, 2020
  • June 1993 LSAT
  • SEC4
  • Q20
1
Reply
Why is A wrong?
Why is A wrong?
andreaskormusis on October 14, 2020
  • June 1993 LSAT
  • SEC4
  • Q23
1
Reply
Why D and not B?
Can you elaborate?
Isabel-Zuniga on August 20, 2020
  • June 1993 LSAT
  • SEC4
  • Q22
1
Reply
Why is B not correct?
Why is B not correct? Thanks
jingjingxiao11111@gmail.com on July 28, 2020
  • June 1993 LSAT
  • SEC4
  • Q22
1
Reply
Help
I understand why rest of the answers are wrong, but how is C right? The answer seems rather extr...
yckim2180 on June 23, 2020
  • June 1993 LSAT
  • SEC4
  • Q16
1
Reply
Option A
I don't understand how A supports the stimulus. Can someone please explain?
Minerva on August 8, 2019
  • June 1993 LSAT
  • SEC4
  • Q11
1
Reply